Proving the range of a piecewise function

Click For Summary
The discussion centers on proving that the range of the piecewise function f:ℤ→ℤ, defined as f(n) = n-1 for even n and f(n) = n+5 for odd n, is equal to the set of all integers, ℤ. Participants express confusion about how to demonstrate that every integer can be represented as an output of the function. It is noted that when n is odd, f(n) results in an even integer, while an even n produces an odd integer, suggesting that both even and odd integers are covered. A clarification reveals that the original claim of ran(f) = 2 was a typo, and the correct assertion is ran(f) = ℤ. The conversation emphasizes the need to show that for any integer, there exists a corresponding n in ℤ that produces it through the function.
ver_mathstats
Messages
258
Reaction score
21

Homework Statement


Allow f:ℤ→ℤ be defined by, for all n∈ℤ

f(n) = {n-1 if n is even, n+5 if n is odd

Prove that ran(f) = ℤ

Homework Equations

The Attempt at a Solution


I am unsure of how exactly to prove this due to the fact now I am working with a piecewise function.
Here is what I have so far:
We must demonstrate that ran(f) ⊆ ℤ and that ℤ ⊆ ran(f).
I know that when n is odd, f(n) is even and that when n is even, f(n) is odd. And now I know I must show that a certain integer is in the range. And I know that if m is odd, m + 1 will be even and that when m is even, m - 5 will be odd, now I do not know where to go from here.
Thank you.
 
Last edited:
Physics news on Phys.org
ver_mathstats said:

Homework Statement


Allow f:ℤ→ℤ be defined by, for all n∈ℤ

f(n) = {n-1 if n is even, n+5 if n is odd

Prove that ran(f) = 2
I don't see how this can be true at all. If you plot a graph of this function, you get two sets of points, one along the line y = x - 1, and the other along the line y = x + 5.
ver_mathstats said:

Homework Equations

The Attempt at a Solution


I am unsure of how exactly to prove this due to the fact now I am working with a piecewise function.
Here is what I have so far:
We must demonstrate that ran(f) ⊆ 2 and that 2 ⊆ ran(f).
Firstly, do I have to split this into cases? I think I must, is this right?
I started working with n-1, if n is even and I got let y ∈ ran(f). By definition of range we can fix x ∈ ℝ such that y = n-1, however since n must be even must we define what even is and substitute it into n? Resulting in y=2k-1. I am unsure of where to go from here if this is correct so far.

Thank you.
 
Mark44 said:
I don't see how this can be true at all. If you plot a graph of this function, you get two sets of points, one along the line y = x - 1, and the other along the line y = x + 5.
There was typo, but it was clarified it was supposed to be ran(f) = ℤ. My apologies, I will fix it right now.
 
ver_mathstats said:
There was typo, but it was clarified it was supposed to be ran(f) = ℤ. My apologies, I will fix it right now.
That makes a lot more sense. Do you see why every integer in ##\mathbb Z## is the image of some other number in this set?
 
Question: A clock's minute hand has length 4 and its hour hand has length 3. What is the distance between the tips at the moment when it is increasing most rapidly?(Putnam Exam Question) Answer: Making assumption that both the hands moves at constant angular velocities, the answer is ## \sqrt{7} .## But don't you think this assumption is somewhat doubtful and wrong?

Similar threads

  • · Replies 16 ·
Replies
16
Views
3K
  • · Replies 2 ·
Replies
2
Views
2K
Replies
5
Views
3K
  • · Replies 9 ·
Replies
9
Views
2K
  • · Replies 5 ·
Replies
5
Views
2K
Replies
5
Views
2K
  • · Replies 6 ·
Replies
6
Views
2K
  • · Replies 14 ·
Replies
14
Views
2K
  • · Replies 3 ·
Replies
3
Views
2K
  • · Replies 3 ·
Replies
3
Views
2K